M11 Betrag eines Vektors: Unterschied zwischen den Versionen

Aus RSG-Wiki
Wechseln zu: Navigation, Suche
Zeile 86: Zeile 86:
 
d) gleich sind keine der Vektoren
 
d) gleich sind keine der Vektoren
  
}}
+
106/7a) <math>|\vec {AB}|=7</math><br>
 +
b) <math>|\vec {AB}|=10</math><br>
 +
c) <math>|\vec {AB}|=17</math><br>
 +
 
 +
106/8 a) <math> |\vec {AB}|=\left | \left ( \begin{array}{c} -8 \\\ k-1 \\\ -1  \end{array}\right) \right| =\sqrt{64+(k-1)^2+1}=9</math> , also <math>65 + (k-1)^2=81</math> --> <math>(k-1)^2 = 16</math> dies liefert zwei Lösungen <br>
 +
<math>k_1-1=-4 \rightarrow k_1=-3</math> und <math> k_2-1=4 \rightarrow k_2 = 5</math><br>
 +
b) k<sub>1</sub>=3 und k<sub>2</sub>=7<br>
 +
 
 +
106/9 Die Dreiecksseiten sind <math>c=|\vec{AB}|, a=|\vec{BC}|, b=|\vec{AC}|</math><br>
 +
<math>c=|\vec{AB}|=\left | \left ( \begin{array}{c} 3 \\\ 0 \\\ -3  \end{array}\right) \right|=3\sqrt 2</math><br>
 +
<math>a=|\vec{BC}|=\left | \left ( \begin{array}{c} 0 \\\ 6 \\\ 0  \end{array}\right) \right|=6</math><br>
 +
<math>b=|\vec{AC}|=\left | \left ( \begin{array}{c} 3 \\\ 6 \\\ -3  \end{array}\right) \right|=3\sqrt {6}</math><br>
 +
Ein Dreieck ist rechtwinklig, wenn der Satz des Pythagoras gilt: <math>c^2+a^2=18+36=54=b^2</math>, also ist das Dreieck ABC bei B rechtwinklig. <br>
 +
Der Flächeninhalt ergibt sich zu <math>A=\frac{1}{2}\cdot 3\sqrt 2 \cdot 6= 9\sqrt 2 </math><br>
 +
Die Umfangslänge ist <math>U=3\sqrt 2 + 6 + 3\sqrt {6}=6+3(\sqrt 2 + \sqrt 6)</math>  }}

Version vom 19. Januar 2021, 12:43 Uhr

Maehnrot.jpg
Merke:

Der Betrag | \vec v | des Vektors \vec v= \left ( \begin{array}{c} v_1 \\\ v_2 \\\ v_3  \end{array}\right) ist  | \vec v | =\sqrt {v_1^2+v_2^2+v_3^2} .

Die Entfernung zweier Punkte P und Q ist der Betrag des Vektors \vec {PQ}.

|\vec {PQ}|=|\vec q - \vec p|= \left | \left ( \begin{array}{c} q_1-p_1 \\\ q_2-p_2 \\\ q_3-p_3  \end{array}\right) \right| =\sqrt{(q_1-p_1)^2+(q_2-p_2)^2+(q_3-p_3)^2}


Bleistift 35fach.jpg   Aufgabe 1

a) Welchen Betrag hat der Vektor \vec v= \left ( \begin{array}{c} 2 \\\ 4\\\ 1  \end{array}\right)?
b) Welche Entfernung haben die Punkte A(2;4,1) und B(-5;3;-1)?
c) Bestimmen Sie den Betrag des Vektors \vec {CD} für C(-5;3,-1) und D(2;4;1).
d) Für welchen Wert von k hat den Vektor \vec v= \left ( \begin{array}{c} -0,3 \\\ k \\\ 0,4  \end{array}\right) den Betrag 1?

a) |\vec v |=\sqrt{21}
b) |\vec {AB}|=3\sqrt 6
c) |\vec {CD}|=3\sqrt 6

d) |\vec v |=\sqrt{0,25 +k^2}=1, also 0,25 + k² =1 liefert k_1=-\frac{\sqrt 3}{2}, k_2=\frac{\sqrt 3}{2}


Maehnrot.jpg
Merke:

Ein Vektor mit dem Betrag 1 heißt Einheitsvektor.
Ein Einheitsvektor wird oft mit \vec e bezeichnet.
Speziell der Einheitsvektor zum Vektor \vec v wird mit \vec {v^0} bezeichnet. Es ist \vec {v^0} = \frac{\vec v}{|\vec v|}


Bleistift 35fach.jpg   Aufgabe 2

1. Gegeben ist der Vektor \vec v= \left ( \begin{array}{c} 2 \\\ 1 \\\ 2  \end{array}\right).
Ermitteln Sie einen Einheitsvektor, der
a) parallel und zu \vec v gleich orientiert ist.
b) parallel und entgegengesetzt zu \vec v orientiert ist.

2. Geben Sie die Einheitsvektoren zu unserem dreidimensionalen Koordinatensystem an.

1. Der Betrag des Vektors \vec v ist |\vec v|=3.
a) \vec {v^0} = \frac{\vec v}{|\vec v|}=\frac{1}{3} \left ( \begin{array}{c} 2 \\\ 1 \\\ 2  \end{array}\right)
b) \vec e = -\frac{\vec v}{|\vec v|}=-\frac{1}{3} \left ( \begin{array}{c} 2 \\\ 1 \\\ 2  \end{array}\right)

2. \vec e_1 =\left ( \begin{array}{c} 1 \\\ 0 \\\ 0  \end{array}\right), \vec e_2=\left ( \begin{array}{c} 0 \\\ 1 \\\ 0  \end{array}\right), \vec e_3 = \left ( \begin{array}{c} 0 \\\ 0 \\\ 1  \end{array}\right)


Maehnrot.jpg
Merke:

Alle Punkte X(x1,x2,x3) im Raum, die von einem Punkt M(m1,m2,m3) die gleiche Entfernung r haben bilden die Oberfläche einer Kugel K.
M ist der Mittelpunkt der Kugel, r der Radius der Kugel.

Für die Punkte X gilt: | \vec {MX} |=|\vec x - \vec m|= \left | \left ( \begin{array}{c} x_1-m_1 \\\ x_2-m_2 \\\ x_3-mp_3  \end{array}\right) \right| = \sqrt{(x_1-m_1)^2+(x_2-m_2)^2+(x_3-m_3)^2} = r

Die Gleichung |\vec {\vec x - \vec m}|=r ist die Vektorgleichung, (x_1-m_1)^2+(x_2-m_2)^2+(x_3-m_3)^2 = r^2 die Koordinatengleichung einer Kugel.


Bleistift 35fach.jpg   Aufgabe 3

a) Geben Sie die Vektor- und die Koordinatengleichung der Kugel mit Mittelpunkt M(2:3:1) und Radius r = 5 an.
b) Welche Lage haben die Punkte O(0;0;0), P(2;0;5), Q(5;6;4) in Bezug auf die Kugel K?

a) |\vec x - \left ( \begin{array}{c} 2 \\\ 3 \\\ 1  \end{array}\right)|=5 und (x_1-2)^2+(x_2-3)^2+(x_3-1)^2=25
b) | \vec {MO} |=|\vec o - \vec m|= \left | \left ( \begin{array}{c} 2 \\\ 3 \\\ 1  \end{array}\right) \right| = \sqrt{14} < 5, also liegt O innerhalb der Kugel.
| \vec {MP} |=|\vec p - \vec m|= \left | \left ( \begin{array}{c} 2-2 \\\ 0-3 \\\ 5-1  \end{array}\right) \right| = \sqrt{25} = 5, also liegt P auf der Kugel.

| \vec {MQ} |=|\vec q - \vec m|= \left | \left ( \begin{array}{c} 5-2 \\\ 6-3 \\\ 4-1  \end{array}\right) \right| = \sqrt{27} >5, also liegt Q außerhalb der Kugel.


Bleistift 35fach.jpg   Aufgabe 4

Buch S. 105 / 1a,g
Buch S. 105 / 2
Buch S. 105 / 4
Buch S. 105 / 5 a,b

105/1a \left | \left ( \begin{array}{c} 2 \\\ 4 \\\ -4  \end{array}\right) \right| = 6
g) 1

105/2a) \left | \left ( \begin{array}{c} -1 \\\ -1 \\\ 0  \end{array}\right) \right|=\sqrt 2
b) \left | \left ( \begin{array}{c} 3 \\\ -3 \\\ 4  \end{array}\right) \right| =\sqrt {34}
c) \left | \left ( \begin{array}{c} -3 \\\ 0 \\\ -2  \end{array}\right) \right| = \sqrt {13}
d) \left | \left ( \begin{array}{c} 5 \\\ -7 \\\ 8  \end{array}\right) \right| =\sqrt {138}
e) | \vec a |=| \vec b| = 3, also \frac{1}{3}|\vec a + \vec b|=\frac{\sqrt 2}{3}
f) \left | \left ( \begin{array}{c} -3 \\\ 3 \\\ -4  \end{array}\right) \right| =\sqrt {34}

105/4 a) (\frac{1}{2})^2 + k^2 + (-\frac{1}{3})^2=1 liefert k^2 = \frac{23}{36}, also k_{1,2}=\pm \frac{\sqrt{23}}{6}
b) k = -1
c) es gibt kein k
d) k_{1,2}=\pm \sqrt 3
e) k_{1,2}=\pm \frac{1}{5}
f) k_{1,2}=\pm \frac{1}{3}

105/5a \left | \left ( \begin{array}{c} -4 \\\ 12 \\\ -3  \end{array}\right) \right| =13, also \vec c=\frac{1}{13} \left ( \begin{array}{c} -4 \\\ 12 \\\ -3  \end{array}\right)

b) \left | \left ( \begin{array}{c} -2 \\\ 1 \\\ 2  \end{array}\right) \right| =3, also \vec c=\frac{1}{3} \left ( \begin{array}{c} -2 \\\ 1 \\\ 2  \end{array}\right)


Bleistift 35fach.jpg   Aufgabe 5

Buch S. 105 / 3
Buch S. 106 / 7
Buch S. 106 / 8
Buch S. 106 / 9

105/3 a) den Betrag \sqrt {14} haben die Vektoren \vec m, \vec q, \vec t , den Betrag 2 haben die Vektoren \vec n, \vec r, den Betrag 2 \sqrt 2 haben die Vektoren  \vec p, \vec s
b) zueinander parallel sind \vec m, \vec t und \vec n, \vec p, \vec s
c) Gegenvektoren sind \vec m und \vec t, sowie \vec p und \vec s.
d) gleich sind keine der Vektoren

106/7a) |\vec {AB}|=7
b) |\vec {AB}|=10
c) |\vec {AB}|=17

106/8 a)  |\vec {AB}|=\left | \left ( \begin{array}{c} -8 \\\ k-1 \\\ -1  \end{array}\right) \right| =\sqrt{64+(k-1)^2+1}=9 , also 65 + (k-1)^2=81 --> (k-1)^2 = 16 dies liefert zwei Lösungen
k_1-1=-4 \rightarrow k_1=-3 und  k_2-1=4 \rightarrow k_2 = 5
b) k1=3 und k2=7

106/9 Die Dreiecksseiten sind c=|\vec{AB}|, a=|\vec{BC}|, b=|\vec{AC}|
c=|\vec{AB}|=\left | \left ( \begin{array}{c} 3 \\\ 0 \\\ -3  \end{array}\right) \right|=3\sqrt 2
a=|\vec{BC}|=\left | \left ( \begin{array}{c} 0 \\\ 6 \\\ 0  \end{array}\right) \right|=6
b=|\vec{AC}|=\left | \left ( \begin{array}{c} 3 \\\ 6 \\\ -3  \end{array}\right) \right|=3\sqrt {6}
Ein Dreieck ist rechtwinklig, wenn der Satz des Pythagoras gilt: c^2+a^2=18+36=54=b^2, also ist das Dreieck ABC bei B rechtwinklig.
Der Flächeninhalt ergibt sich zu A=\frac{1}{2}\cdot 3\sqrt 2 \cdot 6= 9\sqrt 2

Die Umfangslänge ist U=3\sqrt 2 + 6 + 3\sqrt {6}=6+3(\sqrt 2 + \sqrt 6)